Illinois Gifted and Talented Endorsement Practice Questions

Ace your homework & exams now with Quizwiz!

A gifted education teacher is planning programs for the identified gifted and talented students in a middle school. Within the school, there are children of various ages identified with specific gifts in each of the four content areas, as well as in overall intellectual abilities, in the arts, and in leadership abilities. Given these variables, which of the following is most important for the teacher to consider when planning appropriate program options for each of the students? A. Individual programs must be based on one specific area of giftedness. B. Programs must primarily serve those students with specific gifts in each of the four core academic content areas. C. Programs must enable gifted and talented students to work together as a group, with other students, and independently. D. Individual programs must include after-school and summer enrichment opportunities.

C

A gifted seventh-grade student takes a twelfth-grade standardized test designed to measure math aptitude and receives a score equal to the mean for college-bound seniors. When communicating these test results to the student's parents/guardians, the gifted education teacher should emphasize that the student: A. has mastered most of the content of high school math courses. B. is extremely proficient in test-taking skills. C. has exceptional ability in math-related analytical reasoning. D. is capable of passing college-level math courses.

C

Eight-year-old Leah is the sixth child in a family of seven children and is often the one who gets the others out the door and to school on time when their parents have gone to work. At school, she is both an organizer and a diplomat, making sure that every child gets a turn and that playground games run according to the rules. Leah is two years above grade level in math and one grade level above average in language arts. According to this information, Leah also shows signs of giftedness in which of the following areas? A. intrapersonal abilities B. lateral thinking C. leadership abilities D. intuitive thinking

C

Tyrone is a quiet third-grade student with a gift for writing who is educated in a heterogeneous classroom. He spends much of his free time writing plays with well-defined characters and plots, and he shows many of these plays to his teacher, Ms. Moore, who is impressed with their overall quality and depth of character development. Which of the following strategies would be most effective for Ms. Moore to use in providing an appro- priate audience for Tyrone's work? A. having each member of Tyrone's class write a play and providing time to read each one aloud as part of a reader's theater B. suggesting that Tyrone act out his various plays for his family members C. encouraging Tyrone to enter one of his plays in a regional or national contest D. having Tyrone make copies of his plays that can be kept in the classroom library for other students to read

C

Six-year-old Edwin is a musically gifted student who was recently featured as a soloist during a holiday performance of the local philharmonic orchestra. Thereafter, he was interviewed on television and neighbors began to recognize him in the street and talk to him about his brilliant future. At first, Edwin enjoyed the attention, but lately he feels pressured. He hides behind his parents when they go out in public and does not want to practice anymore. His parents are concerned and have requested a meeting with the gifted education specialist. Which of the following strategies would likely be most effective for the gifted education specialist to recommend? A. scheduling small at-home performances so that Edwin becomes comfortable performing in front of people again B. helping Edwin realize that applause and public recognition lead to opportunity but are best taken in stride C. encouraging Edwin to play for his own pleasure and to feel that he is in control of decisions about performing D. helping Edwin realize that great musical ability is the kind of gift that must be developed and shared with others

C Gifted children often feel pressures from parents, teachers, and from their own self-expectations. Edwin has begun to feel external pressures regarding his musicalgifts. His continued musical successes challenge him to keep up with increasingly high expectations. To help reduce Edwin's worries, it is important for his parents to actively focus on encouraging him to perform music for the joy he feels in the music and his own sense of mastery, rather than for the pleasure of his audience, and to emphasize that he can exercise choice about performing in public.

Peter, a gifted six-year-old, was inspired by his older brother's osmosis experiment when he saw how the blue food coloring traveled through a stalk of celery. That night, he dumped a bottle of blue food coloring into his bath to see whether his body transported water the same way and found that it did not. His guardian sent him to school with a note explaining why Peter was temporarily blue from the neck down and asking his first-grade teacher to help address his curiosity about osmosis. Which of the following describes how the first-grade teacher could best adapt the First Grade Core Curriculum in science (Process standard 3: Experiment and Inquiry) to respond in a timely way to Peter's curiosity? A. helping Peter understand that first he will have to develop some necessary skills before undertaking experimentation on plant nutrition and osmosis B. allowing Peter to spend free reading time looking at videos of time-lapse photography on the Internet showing osmosis and photosynthesis C. helping Peter understand what nutrients are and assisting him in devising and performing an osmosis experiment using methods scientists use D. allowing Peter to visit his brother's classroom for a day so he can sit in on the third-grade class's discussion of photosynthesis

C Peter is interested in learning about a concept that he would not usually encounter until a higher grade level. To help Peter explore this topic, it is important to teach him the basic components necessary for understanding what nutrients are and how osmosis works, and then help him construct meaning through creating an osmosis experiment using scientific methods. Students should be encouraged to see science as a process of exploring what is already known about a topic, framing new questions, gathering data, formulating conclusions, and repeating the process as needed. Peter has already begun this process on his own through his experiment with food coloring in the bathtub. It is important for Peter's teacher to encourage these experimental skills in a way that appropriately directs Peter toward framing and experimenting with his questions using authentic scientific methods and vocabulary.

Which of the following is the most important reason to schedule recess for all students in the elementary grades? (A) To provide an opportunity for outdoor fitness pursuits (B) To substitute for physical education classes (C) To foster positive social interactions with others (D) To structure time that students spend playing together

C "Much of what children do during recess, including . . . making choices and developing rules for play, involves the development of social skills." (A) is not correct because recess does not last long enough to practice outdoor fitness skills such as hiking and camping. (B) is not correct because recess is not a substitute for physical education classes that would reinforce fundamental motor skills and movement activities under supervised conditions. (D) is not correct because structured activities would diminish the spontaneity of the children's interactions and potentially decrease the development of social skills that position children to be in charge of themselves.

A gifted student is frustrated and wants to stop working on an independent project. Which of the following is the best first step a teacher can take to enable the student to maintain interest in the project? (A) Agreeing with the student that challenging projects are frequently frustrating (B) Reminding the student that he or she has the abilities needed to do well on the project (C) Helping the student break the project into manageable short-term goals that can be met one at a time (D) Asking the student's parents to assist with the project by locating sources of information the student can use

C Breaking the project into manageable chunks, each of which can be more easily completed than the whole project, is the best answer. "Students may fulfill their goals and objectives by completing projects that match their strengths and interests . . . student and teacher will develop plans, rubrics, timelines, . . ." (A) is not correct because such comments may produce additional frustration and potentially impede progress. (B) is not correct because while affirming ability, providing a framework within which the student can comfortably and diligently demonstrate such ability is preferable and more advantageous. (D) is not correct because inviting the parents to join the effort has the potential to compromise the student's sense of independence.

Which of the following assignments is a young elementary student who is intellectually gifted likely to prefer? (A) Classifying pictures of leaves using a teacher-prepared guide (B) Reading an encyclopedia article on leaves and trees (C) Collecting leaves and deciding on multiple ways to organize them (D) Drawing pictures of leaves found in the neighborhood

C Research on the thinking and learning styles of gifted students indicates that these students tend both to need and to derive considerable satisfaction from activities that involve organizing ideas and objects in a meaningful way, preferably according to principles that they themselves have generated. Gifted children also tend to prefer active exploration over more passive modes of learning. Of the choices listed in the question, only (C) meets all these criteria. For example, collecting leaves is a more active assignment than looking at pictures (A) or reading an article (B). Similarly, finding ways to organize leaves, which involves both generating and applying an organizing principle, is an activity that gifted children would tend to prefer over the activity of simply applying someone else's classification scheme (A). (D) is incorrect because drawing pictures of commonly occurring leaves would offer these children no opportunity to explore, to generate ideas, or to use or derive organizational principles.

Colin is a gifted third grader who breeds mice. During an outdoor recess on the playground, he tries to explain to his classmates what he is doing, but they laugh at him and tell him that breeding mice is dumb. When this happens, he cries and hits his classmates. Which of the following is the most appropriate way for Colin's teacher to find an audience for his creative thinking? (A) Pairing him with an older student who has similar interests (B) Assigning him to work on an open-ended project with classmates (C) Having him prepare a presentation on his project for the class (D) Publishing the results of his research in the district newsletter

C Research suggests "some guidelines for supporting the gifted child . . . [may include arranging for the] child to express his or her overexcitability in a safe environment." (A) is incorrect because unrefined social skills may prove deleterious. (B) is incorrect because parameters with regard to closure may be important. (D) is incorrect because while perhaps a good idea, the better audience would be that of Colin's immediate classmates.

An evaluation of Matthew, a highly gifted 15-year-old student, indicates outstanding achievement in all academic areas. However, observations by several of his teachers as well as his scores on a personality test battery indicate that Matthew has difficulty in several affective areas. He tends to be shy, is excessively concerned with perfection, and strongly prefers working by himself to engaging in group projects. Goals for Matthew that address these concerns are most likely to include which of the following recommendations? (A) Encouraging self-initiated learning in areas of greatest interest to Matthew (B) Developing skill in making judgments using predetermined criteria and guidelines (C) Encouraging participation in academic group problem-solving competitions (D) Developing the habit of reading for pure enjoyment as well as for academic learning

C This question calls for the selection of an educational activity that meets both the cognitive and affective needs of a particular gifted student. Effective IEPs must build on students' strengths as well as address areas in need of remediation. Although Matthew, the student in the example, excels in academic achievement, his shyness and his strong preference for solitary work indicate some weaknesses in his social interaction skills. By encouraging him to participate in academic group problem-solving competitions, he has an opportunity to utilize his strengths (i.e., his knowledge and intelligence) to build up one of his weaker areas (i.e., his difficulty with peer interaction). The other choices are incorrect because they do not address Matthew's strengths and weaknesses. The description of Matthew does not include information about his capacity for self-initiated learning; therefore, (A) is irrelevant and incorrect. (B) is incorrect because Matthew's perfectionism implies that he already excels in the scrupulous application of standards and criteria to his own work and the work of others. (D) is incorrect because although Matthew may benefit from a less task-oriented approach to reading, this would only encourage more, rather than less, solitary activity.

A gifted education teacher is collaborating with teachers from several high schools to develop a new residential summer program in math and science for gifted students who are entering ninth grade. The team is concerned about articulating the summer courses with courses in the students' high schools. Which of the following is the most appropriate way to address this problem? A. limiting participation to students whose high schools are willing to award credit for the summer courses in math and science B. pretesting the students in math and science and assigning students to summer courses based on the results of this assessment C. reviewing the content of the high school programs to determine how they can be augmented by the summer program D. streamlining the curriculum of the summer math and science courses to allow time in the program for enrichment and independent projects

C. Articulation is a process for linking two or more educational systems within a community to foster smooth transitions. Reviewing the content of the high school programs provides information about existing high school courses so that the summer program can be designed to complement what is currently available.

As the Common Core State Standards are adopted throughout the United States, gifted education teachers will need to adapt and modify curriculum, instruction, and assessment. Which of the following additional goals is most important for gifted education teachers to consider first? A. evaluating students' need for curriculum acceleration on a regular basis B. ensuring differentiation of required curriculum is sufficiently challenging C. adhering to the specific guidelines prescribed by the standards D. aligning new benchmarks with the current state curriculum framework

C. Ensuring that differentiated plans for students include the required Common Core content is step one in an effective differentiation plan for gifted and talented students. In addition to incorporating the Common Core Standards, the teacher will still need to address the learning needs of students who are gifted and talented by means such as pre-assessment, incorporation of metacognitive and higher-level thinking strategies, and curricular material above grade-level.

Giving students a broad selection of subject matter encourages _____ thinking. A. iconoclastic B. repetitious C. divergent D. conversant

C. Gifted students should be given as much freedom with their curriculum as possible.

Elements of the theory of multiple intelligences are: A. Everyone learns in the same way B. Some people are smarter than others C. Some people learn primarily by hands-on projects D. All of the above

C. Multiple intelligences is a theory developed in 1983 by Dr. Howard Gardner, a Harvard professor of education. He believes everyone learns according to one of eight intelligences. Adopting and utilizing his theory offers a variety of teaching tools to enhance lectures and create activities that spur the imagination and expand learning opportunities. A well-prepared teacher with an interactive lesson plan already uses Dr. Gardner's theory. They know, through experience and observation that students learn in different ways. Teachers lecture (verbal-linguistic intelligence or word smart) requiring students to think conceptually and link facts together (logical-mathematical intelligence or number and reasoning smart) and use pictures, charts and other props (visual-spatial intelligence or picture smart and, depending upon the material, music-rhythmic intelligence or music smart) during the presentation. Many lesson plans include hands-on projects (bodily-kinesthetic intelligence or body smart and, depending upon the subject, naturalist intelligence or nature smart). Asking thought-provoking questions that encourage lively class discussions (interpersonal intelligence or people smart and intrapersonal intelligence or self-smart) are classroom staples.

Based on Karl's academic profile, which of the following adaptations would be most appropriate for Karl in the areas of science and social studies? A. designing the learning environment for Karl to include computer-based instruction B. requiring Karl to use manipulatives during problem-solving activities C. placing Karl into a differentiated group that requires students to use higher-order thinking skills D. requiring Karl to demonstrate understanding by constructing three-dimensional models related to content topics

C. Since Karl shows an excellent grasp of abstract concepts in social studies and science, he is ready to develop higher-order thinking skills of analysis, synthesis, and evaluation as reasoning tools to help him construct meaning and solve problems at a level which challenges his abilities. Thus, a differentiated group focusing on the use of higher-order thinking skills is the most appropriate adaptation for Karl.

Ms. Edwards, a newly hired third-grade teacher, is enjoying her lively mixed-ability class. Sometimes however, there are conflicts arising between Justin, a verbally gifted student, and other students who often take offense when Justin uses words that they do not understand or that they believe are just insults directed at them. So far Ms. Edwards thinks that she has been able to handle these situations effectively but she would like to read more about these types of issues to better understand what triggers them and how to prevent them. Which of the following online resources would provide the best starting point for her research? A. Davidson Institute for Talent Development B. Gifted Child Quarterly C. Supporting Emotional Needs of the Gifted D. National Institute for Mental Health

C. Supporting Emotional Needs of the Gifted (SENG) "seeks to inform gifted individuals, their families, and the professionals who work with them, about the unique social and emotional needs of gifted persons." To that end, they offer web seminars, an articles library, parent support and training, as well as an annual conference focusing on the emotional and social issues of the gifted and talented.

Mr. Allen is a new middle school principal who is interested in knowing more about how to incorporate the Schoolwide Enrichment Model (SEM) to develop more opportunities for students who are gifted and talented. He learns both the school and community provide various Type 1 enrichment activities for students who are gifted and talented, including field trips to the local museum. However, he feels the school could do more to provide these students with Type 2 enrichment activities. Which of the following activities is an example of a Type 2 enrichment activity? A. collecting money to buy books for preschools in impoverished areas B. attending a minicourse on prominent local historical figures C. studying primary resources from the era represented in a historical novel D. listening to an interview with an expert on Shakespeare before watching a performance

C. The Schoolwide Enrichment Model, or SEM, developed by Joseph Renzulli and Sally Reis of the Neag Center for Gifted Education and Talent Development, divides enrichment activities into three types. Type 1 consists of activities that expose students to people and ideas that they might not encounter during the typical school day, such as listening to an interview with an expert on Shakespeare. Type 2 is defined by activities that promote the use of materials that help students to develop problem-solving and creative skills as well as the ability to use and evaluate advanced research materials. Studying primary resources that reflect the content of an historical novel allows the student to develop greater insight into the novel's historical context.

Many of the students at Somerset Elementary School have limited English proficiency. The new gifted-education consultant for the district observes that none of the school's English language learners are currently included in the district's gifted program. The consultant wants to ensure that there is an equitable assessment process in place. Which strategy would likely be most effective in identifying high-ability students among this student population? A. instituting ongoing evaluation of potential giftedness based on students' classroom participation B. conducting focused IQ testing in multiple subject areas not requiring English language mastery C. using multiple criteria beyond academic achievement to identify students' strengths and potentials D. intensifying English language instruction to increase the reliability of standardized test results

C. This item requires examinees to apply knowledge of nonbiased and equitable processes and procedures for identifying and placing students who are gifted and talented (e.g., collecting multiple types of assessment information, using locally developed norms or assessment tools in students' home languages, using nonverbal assessments, using culturally sensitive checklists, eliciting evidence of students' interests and potentials outside the classroom setting, and providing testing accommodations to students with disabilities). Some gifted education programs use the results of standardized tests as the primary criteria for identifying students for the program. For English language learners, standardized tests are less than reliable indicators of giftedness due to language differences. More reliable data about potential giftedness in English language learners can be obtained through a combination of several different assessments, such as student portfolios, dynamic performance-based assessments, and parent/guardian interviews. This approach helps mitigate the effects of language differences in the identification process.

In school districts populated largely by students from diverse cultural and ethnic backgrounds, teachers should follow which of the following procedures when identifying students for a gifted enrichment program that is based on individual students' strengths and interests? A. To ensure commitment to the program, primarily consider students who, during a personal interview, convincingly express their interest. B. Use assessments that measure attributes and behaviors of giftedness as they may manifest in these groups of students. C. Mainly consider students who receive parent/guardian nominations to ensure support within their community for the students' participation. D. Select students who have main- tained a grade-point average of 90 percent or higher for at least three consecutive semesters.

Correct Response: B. Researchers have recently been considering how to identify giftedness in students from diverse backgrounds. In the past, a disproportionately low number of students from some cultural and ethnic minority backgrounds have been identified for receiving gifted services. Researchers have concluded that traditional assessment instruments, such as IQ tests and standardized achievement tests, may not be the best indicators of giftedness for these groups. Such tests do not measure aspects of giftedness such as creativity and leadership ability. Alternative forms of assessment, such as performance assessments, teacher referrals, and case studies, have been shown to be more successful in identifying students who would benefit from gifted education but who might not otherwise be targeted for such programs.

A number of gifted high school students are in a Language Arts class. Is their AP teacher obliged to differentiate the program of study for the gifted students? a. No; if the teacher is already running a student-centered classroom, allowing students choices in assignments, and using flexibility in student grouping, the teacher is already differentiating. No further modification is required b. No. All students in Advanced Placement courses are, by definition, gifted. Therefore, differentiation is already taking place c. No. The teacher can choose to differentiate the program, but by the time gifted and talented students have entered high school, they have internalized sufficient strategies to challenge themselves and independently promote their own education d. Yes. If the teacher is already running a student-centered classroom, allowing choices in assignments, and using flexibility in student grouping, the teacher is already differentiating. The teacher has received additional training to allow him or her to continue to modify classroom assignments to more profoundly challenge the gifted students

D

During the second quarter of the school year, a kindergartner seems bored and easily distracted, although he answers all of the teacher's questions and completes work quickly and accurately. At recess, he devises strategy games with detailed explanations that his peers do not under- stand. When the teacher arranges to have him tested for admission to the school's gifted and talented program, the student shows advanced ability in both verbal and quantitative reasoning. Which of the following gifted programming options would be most appropriate for this student at this point in the school year? A. independent study B. subject-specific differentiation C. technology-enhanced instruction D. early-entrance grade acceleration

D

Gifted students with learning disabilities can often be distinguished from other students with learning disabilities, because when gifted students with learning dis- abilities are faced with a challenging task that aligns with their strengths and interests they most often: A. attempt to complete the task as quickly as possible. B. become distant from and uncom- municative with the teacher and classmates. C. tell the teacher they are unable to complete the task. D. try a variety of problem-solving and compensatory strategies to reach their goal.

D

Lisa is a fifteen year old who is gifted in science and who is particularly interested in astronomy. Her high school currently offers one science elective in astronomy, and she is currently enrolled in this class, which ends halfway through the school year. Which of the following recommendations, given by Lisa's gifted education teacher, would best promote further academic advancement in her area of giftedness? A. developing an after-school astronomy club for middle school students B. organizing and conducting a star- gazing event for members of the local community C. offering to mentor peers for projects involving astronomy and other science topics D. enrolling in an online astronomy course offered through a college or university

D

Marie is an eighth-grade gifted student who has a demonstrably well-developed interest in artificial intelligence, and she is considering majoring in this field of study in college. Which of the following learning characteristics would most strongly indicate that a mentorship would be an appropriate program option to support her needs and talents? A. She consistently completes assign- ments on time. B. She is comfortable in competitive situations. C. She shows a lack of patience when working with classmates on projects. D. She is focused and intense when learning.

D

Brock is a fourteen-year-old highly gifted tenth grader. Brock is gay and recently came out to his family and teachers. Though family members and teachers have been supportive, Brock is determined that his peers not become aware of his sexual orientation. He is popular, participates actively in athletics, and excels in all academic areas. Brock's determination to maintain a "straight" persona among his peers, a behavior common among sexual minority youth, is most likely attributed to a: A. lack of effective coping skills. B. keen sensitivity to others' emotions. C. strong sense of personal identity. D. fear of being shunned or harassed.

D Being both gifted and gay presents unique challenges and it can be difficult for a student to find a peer group who identifies with both of these characteristics. Although gay and lesbian rights are becoming more recognized than in the past, students who identify with these groups still experience discrimination and isolation based on their sexual orientation. By participating in sports and choosing to be known as heterosexual at school, Brock identifies with a larger peer group and reduces his chances of experiencing discrimination and isolation.

Jaleysia is an eighth-grade student gifted in the visual arts. She has taken art lessons since she was in kindergarten and has won several local art contests for her paintings. In her academic classes, Jaleysia learns new material very quickly and participates actively in class discussions, but she consistently underperforms on written tasks and projects, saying that she finds research and writing tedious. Her science teacher consults with the school's gifted education specialist regarding possible differentiation designs for an upcoming cell biology research project. Which of the following assignments would provide the best differentiation for Jaleysia? A. Using your textbook and class notes, create a poster of key facts about cells and their development. B. Research a current topic or issue related to cell biology not discussed in class, and create a brochure to inform the class about the topic. C. Develop a slideshow presentation to illustrate important vocabulary words related to cell development. D. Using your textbook and class notes, design a mural that will help others in class remember key concepts in cell development.

D The science teacher clearly wants to differentiate the lesson to make it more motivating and engaging for Jaleysia. By asking Jaleysia to design a mural that would integrate key concepts in cell development, the teacher can promote Jaleysia's engagement with the lesson content while allowing her to continue to develop and explore her areas of interest. The differentiated assignment will challenge her by having her integrate her new knowledge of cell development into the production of a mural that will be useful to help educate those in her environment as well.

Colin is a gifted third grader who breeds mice. During an outdoor recess on the playground, he tries to explain to his classmates what he is doing, but they laugh at him and tell him that breeding mice is dumb. When this happens, he cries and hits his classmates. Which of the following challenges often associated with gifted students best explains why Colin is behaving as described? (A) He is very intense and demands perfection in himself. (B) He has an unusual hobby and enjoys making discoveries. (C) He rebels against routine and predictability. (D) He is supersensitive to any form of criticism.

D According to one source, giftedness has been described as "overexcitabilities [which] can . . . cause problems with social interaction, daily functioning, and school experiences . . . [and] may be expressed [as] . . . impulsive actions," that in turn may cause others to criticize the individual displaying these behaviors. (A) is incorrect because intensity and demands of perfection may not be consistently characteristic of the gifted. (B) is incorrect because while this may be so, having unusual hobbies and the joy of making discoveries is not uniquely characteristic of the gifted. (C) is incorrect because while the gifted may demonstrate rejection of routine and the predictability of circumstances and events, such behavior is not considered to be characteristic of giftedness.

What is one of the primary goals of a professional organization? (A) Mandating a national curriculum (B) Controlling publication of articles related to the profession (C) Enforcing standards for the practice of the profession (D) Providing leadership for the improvement of the profession

D It cannot mandate what is taught, nor can it control what is published about it. It cannot enforce standards, though it can support and endorse them.

Which of the following is most likely to be a primary consideration in the selection of a curriculum model for gifted students? (A) The model should be focused primarily on content that is appropriate for gifted students, leaving process-related decisions up to the individual teacher. (B) The model should be a feasible adjunct to the regular curriculum. (C) The model should maximize opportunities for the integration of gifted and regular students in instructional activities. (D) The model should provide multiple paths to reach specified goals.

D When educators select a curriculum model for gifted students, a number of principles should guide them. One of these principles involves flexibility in reaching specified goals. This factor is important in allowing teachers to be responsive to the diverse needs, interests, and talents of their students, as well as to the particular constellation of resources that may be available in a given situation. (A) is incorrect because a curriculum for gifted students should define not only the type of content that is most appropriate but also how to present this content in ways that will best challenge the particular intellectual strengths and interests of gifted students. Concerning (B) and (C), the feasibility of linking the gifted curriculum to the regular curriculum, or the gifted students to the regular students, may be an issue to consider in some particular circumstances. However, because these considerations have nothing inherently to do with the question of how best to serve gifted students, they should not be the guiding principles in curriculum development.

Eight-year-old Stephen lives in a socioeconomically diverse area. He is profoundly gifted in mathematics and has exhausted the resources of the local elementary, middle, and high schools. He is two years above grade-level in science and English language arts, both of which he takes with the fifth-grade class. He is socially reticent and uncomfortable with athletic activities. Which of the following strategies would be most appropriate for the school district's gifted education specialist to propose to address Stephen's needs at this time? A. following his curiosity where it leads by doing an unstructured independent study in mathematics B. stretching his mathematical skills by taking a physics class at the local high school C. taking a break from formal mathematics instruction in favor of activities developing his social skills D. advancing his mathematics skills using online course materials under the supervision of a teacher

D. An elementary school child who is profoundly gifted in mathematics and has exhausted all relevant local resources must, first and foremost, have his educational needs met in an appropriate way. Online instruction with active teacher supervision provides opportunity and ongoing academic guidance as well as personal interaction and support. Social reticence and a lack of interest in sports are aspects of Stephen's personality at this stage in his development, not areas of need to be addressed by the gifted education specialist.

In an effort to encourage Karl to interact more with his classmates, Karl's teacher has begun teaching the class cooperative games. Using cooperative games is an effective strategy for: A. encouraging gifted children to rely on each other to achieve a common reward. B. preparing gifted students for structured cooperative learning activities in academics. C. helping gifted students learn to win and lose gracefully. D. providing gifted children with a nonthreatening context for practicing social skills.

D. Cooperative games would provide a safe and stimulating environment for Karl to interact with his classmates. By emphasizing participation and challenge, and by enabling children to think together as a team where everyone has input and shares ideas, these games provide social skills practice in a non-threatening context. Cooperative games encourage children not only to work together, but also to be leaders, to share, and to help others.

Research has shown that gifted students are better than most students at discovery learning because gifted students: A. can summarize information more easily than most students. B. prefer not to follow directions prescribed by the teacher. C. have greater creativity than most students. D. have many schemas to make connections and see patterns.

D. Discovery learning is personal, internal, and constructivist. It is inquiry-based and requires learners to draw on their own experiences and prior knowledge to place new learning in a meaningful context. Gifted students' reasoning abilities provide them with schemas, or arrangements of knowledge, that help them understand new information and help them make connections and discern patterns.

Questions to ask when developing a performance contract could include: A. What information will be included? B. What research resources will be used? C. How long will the project take to complete? D. All of the above

D. In order for a performance contract to be a learning experience, the guidelines for writing one should be very general. The teacher can either give the student a written list of suggestions or better yet, discuss them one-on-one. Some questions that might be used: What work items are you planning to include? Where you will find the necessary data? Personal reference books? The Internet? The library? Do you have additional sources? How long will it take to outline a plan; research the topic; finish the project? What criteria should be used to evaluate the finished product?

Tom is a five year old in a full-day kindergarten program. He often becomes very enthusiastic about a topic and interrupts the teacher with additional information or comments. Tom's behavior sometimes annoys his classmates, and he often delays instruction with detailed explanations of various topics. Which of the following actions would be most effective for the teacher to take in addressing Tom's behavior? A. explaining to Tom that his behavior is disruptive to others and asking him not to speak out in class B. providing Tom with a seat close to the teacher during instructional activities to facilitate redirection of his behavior C. requesting that Tom be placed in a classroom with a higher teacher- student ratio to provide him with more individualized attention D. providing Tom the opportunity to study topics in depth and share his expertise with classmates

D. Providing Tom the opportunity to study topics in depth and share his expertise with classmates represents an academic and behavioral intervention that nurtures his ability to probe for details and construct explanations. At the same time, this option is the most effective for addressing Tom's behavior, because it provides him with a specific forum for sharing his expertise and encourages him to do so in an appropriate and positive context that should discourage his frequent interruptions.

According to the Javits Act, gifted and talented students are described as those who: a. Have been assessed as having intelligence at least 30% above the national average, and are not receiving services or activities to develop those capabilities b. Are in the top 5% of their age group in terms of intellectual, creative, artistic, or leadership areas, or in specific academic fields c. Have achieved high accomplishment in intellectual, creative, artistic, or leadership areas or in specific academic fields, and who have not been recognized and honored in an appropriate manner d. Have the capability for high accomplishment in intellectual, creative, artistic, or leadership areas or in specific academic fields, and who need services or activities not ordinarily provided by the school to develop those capabilities

D. The Jacob Javits Gifted and Talented Students Education Act (Javits) is a federal program that addresses the needs of gifted and talented children. The purpose of the Act is to coordinate programs of scientific research, projects, pioneering approaches, and the like to enhance schools' abilities to foster the educational needs of gifted and talented students.

According to research conducted by Carol Dweck, which of the following approaches is most effective for encouraging students who are gifted and talented? A. reminding the students that because they are smart, they will do well on upcoming projects B. delaying gratification to help the students learn to work for longer periods of time C. identifying tasks for the students that are just beyond their current abilities D. acknowledging the students' hard work and effort in meeting identified learning goals

D. The research of Carol Dweck, PhD, supports the idea that encouragement for hard work and effort, mastery orientation, rather than praise for fixed traits and actions, a performance orientation, actually helps students to embrace challenge and strive to do their best, even when faced with difficult tasks. Dweck's research distinguishes between a fixed mindset, for instance, a focus on innate talent, and a growth mindset or a focus on dedication and resilience. Her research reveals a meaningful difference between creating student dependency on expressions of approval from others and fostering inner direction and a sense of ownership and responsibility for their efforts.

Ms. Murphy, an elementary gifted education teacher, offers workshops each month to the families of students within the school's gifted education program. The workshops focus on different topics related to gifted students and their families. This strategy would be most effective for: A. assessing parent/guardian interest in the gifted education program. B. increasing the likelihood that families of gifted students will participate in schoolwide events. C. measuring the success of the gifted education program. D. providing opportunities for families to come together as advocates for their children.

D. The workshops offered by the gifted education teacher will empower families as advocates for their children by raising their awareness of what they can do to assure that their children's educational experiences in and out of school match the children's needs and abilities. Also, the workshops will foster communication within the group, and with teachers and other school personnel, on behalf of the gifted students.

A science teacher is collaborating with the gifted education specialist to write appropriate science curricula for moderately to highly gifted elementary school students. Which of the following instructional approaches would be most effective for meeting a variety of individual learning needs in science? A. a technology-based program that emphasizes high-level content and depth B. a module-based curriculum that includes pre-assessments to determine student entry level C. a higher-level secondary school curriculum modified to meet elementary school time and staffing constraints D. a range of learning objectives and experiences that emphasize inquiry, higher-level conceptual thinking, and problem-based learning

D. There is considerable difference between the abilities and learning needs of a moderately gifted child and a highly gifted child. The teacher of both moderately and highly gifted students will encounter a range of individual learning needs, skills, and aptitudes within the gifted population. By selecting learning experiences that emphasize inquiry, higher-level conceptual thinking, and problem-based learning, the content and learning targets can be appropriately differentiated according to student need, interest, and ability.

Which of the following procedures would be most appropriate for a gifted education teacher to follow when evaluating a student with a hearing loss for gifted services? A. delaying assessment for gifted identification until the student has acquired sign language B. observing the student's performance in a classroom of peers who are deaf or hard of hearing C. basing recommendations on standardized tests that measure achievement rather than general intelligence D. evaluating the student's capabilities in an environment that stimulates cognitive performance

D. While modification of assessment may be necessary, the presence of a physical disability does not in any way lessen the importance of evaluation and identification of students who may be appropriate for gifted services, or suggest a need to delay such evaluation. If anything, the existence of a physical disability makes it particularly important that assessment take place in an environment in which the student has every opportunity to demonstrate the full range of her or his cognitive abilities.

There have been a number of belief systems regarding the nature of intelligence. According to Spearman, intelligence is a measurable cognitive capacity to which a constant number can be assigned; an individual who is intelligent in one area is equally intelligent in others. For Gardner, there are several distinct types of intelligences, and an individual might excel in one area but not another. Sternberg theorizes intelligence is reflected in how successfully an individual handles the changing environment through time. He located cognitive processes in three processes; performance components, knowledge-acquisition components, and: a. Metaphoric components b. Analytic components c. Supercomponents d. Metacomponents

D: Metacomponents. Sternberg theorizes intelligence is reflected in how successfully an individual handles the changing environment through time. He located cognitive processes in three processes; performance components, knowledge-acquisition components and metacomponents. Metacomponents are self-aware processes that contribute to making decisions and solving problems; they can be seen as instances of the mind being aware of the mind.

Mr. Cabrera is a gifted education teacher who teaches gifted and talented elementary school students in a pull-out program. In which of the following situations is he acting as a facilitator? A. Mr. Cabrera meets with other gifted education teachers in the school district to develop a new summer enrichment program. He is currently investigating possible locations and resources for the program. B. Mr. Cabrera's fourth-grade students are involved in independent research projects. He meets with students on a weekly basis to discuss their progress and provide them with resources and guidance as needed. C. Mr. Cabrera attends a school board meeting to present information about the need to increase services for gifted students in the district. He cites findings from current research in the field to support his argument. D. Mr. Cabrera has recently read several articles about a new instructional technique to use with gifted and talented elementary students. He decides to try the technique with his third-grade students on a trial basis and analyze the results.

Option A describes a situation in which Mr. Cabrera is working together with other gifted education teachers to develop a new program. In this role, Mr. Cabrera is collaborating with others to reach a goal. Therefore, he is serving in the role of a COLLABORATOR rather than as a facilitator. Option B describes a situation in which Mr. Cabrera's students are involved in independent research projects. He meets with his students weekly to discuss their progress and to provide support in the form of resources and guidance. During these projects, Mr. Cabrera provides a learner-centered environment in which he serves as a support and guide rather than as an authority figure; he enables students to pursue their own learning goals. Therefore, he is serving in the role of a FACILITATOR. Thus option B would be the correct response to this item. Option C describes a situation in which Mr. Cabrera attends a school board meeting to present information about the need to increase services for gifted students in the district. He presents an argument in favor of expanding services for gifted students. Therefore, he is serving in the role of an ADVOCATE rather than as a facilitator. Option D describes a situation in which Mr. Cabrera has read several articles about a new instructional technique to use with gifted and talented elementary students. He decides to conduct action research by using the technique with his students on a trial basis and analyzing the results. Therefore, he is serving in the role of a RESEARCHER rather than as a facilitator.

Carlos, a sixth-grade student, exhibits originality in both oral and written expression, and he is most challenged by tasks that require him to take an alternate point of view. His teachers often describe him as an individual who does not mind being different from the crowd. His grades fluctuate depending on the class and the topic. The characteristics Carlos exhibits are most similar to those of a student gifted in: A. general intellectual ability B. leadership C. a specific academic ability D. creative thinking

The characteristics exhibited by an individual who is gifted in general intellectual ability include the following: learns rapidly, processes information in complex ways, is excited about new ideas, is a self-starter, is inquisitive, and is an avid reader. The characteristics exhibited by an individual who is gifted in leadership include the following: assumes responsibility, has high expectations for self and others, demonstrates good judgment in decision making, likes structure, and is self-confident and organized. The characteristics exhibited by an individual who is gifted in a specific academic ability include the following: acquires basic-skills knowledge quickly, has high academic success in a special-interest area, is well-read in a special-interest area, memorizes easily, and has advanced comprehension ability. The characteristics exhibited by an individual who is gifted in a creative thinking include the following: exhibits original thinking in oral and written expression, is challenged by creative tasks, is an independent thinker, and does not mind being different from the crowd. According to the profile presented about Carlos, he exhibits the characteristics of a student who is gifted in creative thinking. Thus option D, creative thinking, would be the correct response to this item.

An advantage of curriculum compacting compared to grade advancement for moderately gifted students is that compacting: A. allows for differentiation across curricular areas. B. eliminates social stigma because it is undetectable to classmates. C. promotes leadership and social skills of gifted students. D. encourages gifted students to excel in all curricular areas.

A

What is the best assessment vehicle for students' creative writing? A. Portfolios B. Pop quizzes C. Recitations D. Interviews

A. A writing portfolio is a way for students to take a holistic look at their coursework.

The elements of a performance contract are: A. A promise from the student B. A timeline C. Strict discipline D. All of the above

B. A performance contract is a written agreement between one student or a group of students and the teacher about a specific activity. The contract clearly states the goal, explains the activity, establishes a timeline and describes who will do what and how it will be done. Sometimes the agreement explains the criteria to be used to evaluate the finished product. This tool helps students learn how to plan a project and utilize their time more efficiently. Not only can the completed project be graded, but also the performance contract itself can be evaluated. The teacher should assess the student's participation in setting up the contract, willingness to compromise when necessary and general attitude about the concept and the process. Performance contracts can be a great learning experience for students by teaching them how to plan and prioritize. When used for a group project, it requires collaborating on the details of the contract and working with other students to reach a common goal; two activities gifted students sometimes have difficulty managing.

Who asserted that gifted children are likely to exhibit high levels of psychomotor energy? A. Piaget B. Dabrowski C. Erikson D. Montessori

B. Dabrowski identified five areas of common overexcitability in gifted children: psychomotor, sensual, emotional, intellectual, and imaginational.

Strategies to keep the attention of gifted students may include: A. Require rote memorization B. Avoid lag time in lesson plan C. Test frequently D. All of the above

B. Gifted students have active imaginations and their brains are always "on." To prevent boredom and keep their attention, a teacher must avoid lag time by preparing a lesson plan that fills the entire class period. Moving around, varying voice tone and timber and presenting at a brisk pace all contribute to keeping students' focused on the subject matter. Asking thought-provoking questions and posing interesting scenarios requires critical thinking, stimulates discussion and encourages them to reason things out for themselves. Throwing out comments and quick questions that only require a one or two word answer keeps them involved. Breaking the class into small groups, giving each group a specific task and having each one present their findings to the whole class provides a challenging change of pace, keeps them involved and helps students learn to work as a team. This is especially important for gifted students because some of them have difficulty relating to and working with others; so they need practical experience in this area of their development.

Which of the following is NOT typically greater among students who score above average on standardized tests? A. Social adjustment B. Poise C. Organization D. Physical size

C. Academically advanced students also tend to be more physically and socially developed than their peers.

Ms. Shah, a high school gifted education teacher, gives her students in advanced science an essay test that asks them to explain how to apply problem-solving skills to a science issue of their choice. She expects student responses to be related to ecology, the focus of their first- semester study, and she is impressed by the rich array of issues raised by the students in their essays. Because of the students' expressed interest in and depth of thought about diverse issues, Ms. Shah decides to replace the whole-group single- issue investigation she had planned for the second semester with guided independent study projects. Which of the following best explains the reason for Ms. Shah's decision? A. She wants students to sharpen their critical-thinking ability by examining multiple viewpoints. B. She evaluates group dynamics and believes that students will create diverse, productive outcomes. C. She wants students to learn to organize and cross-reference data without outside supervision. D. She uses formative assessment as a tool for revising curriculum to meet students' needs.

D

When a gifted student is involved in independent study, what is the major responsibility of the student's teacher or mentor? (A) Identifying appropriate resources to support the student's learning (B) Analyzing data and drawing conclusions from the student's projects (C) Determining an appropriate format for the presentation of results (D) Helping the student focus on appropriate topics for study

D An important aspect of independent study for a gifted student is ensuring that the student selects appropriate topics to serve as a focus for his or her project. However, because students often have difficulty determining the appropriate level of specificity for a study topic, the teacher has a major responsibility to help students focus their study topics in such a way that a productive project is possible, given limitations of time, of resources, and of the student's intellectual maturity (D). In specific situations, a teacher may sometimes decide to assist a student in identifying resources (A) in analyzing data and drawing conclusions (B), or in determining an appropriate format for presentation of results (C). However, since none of these activities is in all cases a major responsibility of the teacher, (A), (B), and (C) are incorrect.

Students identified as gifted and talented can demonstrate serious lack of motivation that hinders their performances. Some of the factors contributing to underachievement are low self-efficacy, not finding the class interesting, not finding the class useful, a student's lack of self-regulation skills, and a. When the work is too challenging b. If the student is ostracized and unwilling to participate in class c. Because the child is so bright, the family has treated him or her with indulgence. As a result, the child expects to be treated as special d. The work isn't challenging enough

D Gifted and talented students learn best when they are interested in the material, allowed to explore it independently and are challenged by it. They are not motivated by grades or products so much as by the learning process.

Which of the following best explains why a middle school teacher of the gifted has students read stories written by Arthur Conan Doyle and observe that his detective, Sherlock Holmes, is particularly skilled at drawing conclusions from observing seemingly irrelevant details? (A) To help students understand the limitations of logical reasoning (B) To encourage students to develop intellectual and emotional regulation (C) To sensitize students to contradictions and paradoxes in the world (D) To teach students that examining detail may help to ascertain meaning

D Having Sherlock Holmes model the importance of examining what is often not noticed will aid students in analyzing, not only literature, but life situations in general. According to the Cuesta College Web site, "Drawing conclusions refers to information that is implied or inferred . . . never clearly stated. " In addition, "Making inferences means choosing the most likely explanation from the facts . . . derived from observation. . . ."In fact, the Web site/author continues "knowledge [is] observable phenomena—the result of detailed observation." And, "In drawing conclusions [making inferences], you are really getting at the ultimate meaning of things— what is important, why it is important, how one event influences another, how one happening leads to another. Simply getting the facts in reading is not enough—you must think about what those facts mean to you." (A) is not correct because there is a different instructional strategy used in the development of logical reasoning. (B) is not correct because the development of intellectual and emotional regulation requires a different process. (C) is not correct because sensitization strategies would be uniquely different.

A gifted education teacher believes in a social-psychological approach to developing creativity in students. According to this approach, gifted students are more apt to demonstrate creativity when the teacher encourages them to develop: A. intrinsic motivation. B. cooperative-learning skills. C. self-discipline. D. critical-thinking skills.

A

Which of the following is the most important reason for using multiple forms of assessment when screening for giftedness in children with disabilities? A. The nature and impact of disability-related developmental delays may affect performance on a standard instrument at a given time. B. Stereotypical attitudes about twice exceptional individuals may affect the way in which standard assess- ments are administered. C. Educators' lack of training in addressing dual exceptionalities may affect educators' ability to evaluate individual performances. D. The shortage of specific identification instruments for those with disabilities may affect the interpretation of scores.

A

A gifted education teacher is developing a test to informally assess a third grader's divergent thinking. Which of the following activities would be most appropriate for the teacher to include on this test? A. List as many uses of a pencil as you can. B. Draw a picture of the school's gymnasium. C. Write all of the factors of 24. D. Select your two favorite kinds of fruit juice.

A Divergent thinking typically occurs in a spontaneous, free-flowing manner, by brainstorming many ideas in a short amount of time and in an emergent cognitive fashion. Asking a student to list as many uses of a pencil as possible would be a means of assessing a student's level of divergent thinking.

A gifted education specialist has been asked by the principal to create a new differentiated program for fourth-grade social studies. There are currently eight different fourth-grade classes in the school. Most of the teachers are unfamiliar with differentiated instructional strategies for gifted students. Which of the following strategies should the specialist employ to ensure the successful implementation of the program? A. providing in-service training about the characteristics and needs of gifted students and how to meet these needs, as well as offering ongoing support B. distributing a copy of Oklahoma state law regarding the Education of Gifted and Talented Students and reminding teachers of their responsibility to remain in compliance C. offering Professional Development Points for attending after-school study groups on differentiation for gifted students D. making a presentation at a full faculty meeting that highlights how differentiation for gifted students will help improve the school's overall test scores

A Research indicates that teachers who have received ongoing training in gifted education are more likely to promote the development of students' higher-level thinking skills, allow for greater student expression, consider individual student strengths and weaknesses, and provide a wider variety of learning experiences to challenge their students. This vital expertise benefits all students but requires high-quality professional development, time, materials,andcontinuedsupport. Byprovidingappropriatetrainingandmaterials,thegiftededucationspecialistwill ensure that the new differentiated program will be successful with both teachers and gifted students.

Soledad is a gifted sixth grader who has developed a propensity for blurting out answers during class discussions. Which of the following is the best way for the teacher to help Soledad control this impulse and thereby ensure that all students in the class have an equal opportunity to participate? (A) After consulting with the school psychologist, the teacher will model strategies such as talking to oneself. (B) Asking only open-ended questions and giving students 10 to 15 seconds of wait time to think about their answers before calling on any one to respond (C) Ignoring all attention-getting behaviors such as hand-waving in favor of selecting a name at random from a prepared set of name cards (D) Preassigning questions to groups of 3 to 5 students and having them make notes before presenting their responses to the class

A "When psychologists are asked to become involved with gifted students, usually the referrals have to do with . . . behavioral issues such as . . . impulse control difficulties . . . [and they] can . . . assist with educational planning. . . ."Research claims "Talking to ourselves in [an] 'inner voice' actually helps us exercise self-control and prevents us from making impulsive decisions." Importantly, "Such strategies establish neural connections between survival instinct impulses and the executive brain's understanding of limits and boundaries." (B) is not correct because while wait time may be a valuable intervention, modeling preferred behavior is best. (C) is not correct because ignoring inappropriate behavior may be beneficial, or it may increase frustration; modeling correct behavior will likely foster greater impact. (D) is not correct because it does not address the impulsive behavior in the classroom.

Which of the following principles is most appropriate for guiding the development of a gifted education program? (A) Gifted students should be given multiple educational opportunities to realize their potential to the fullest extent possible. (B) The primary aim of instruction for gifted students should be to provide students with learning experiences that help translate the affective domain into thoughts and actions. (C) Gifted students should be instructed in homogeneous groupings to the maximum extent possible. (D) Education for the gifted should be administered as a separate program from the regular education program to maximize available funding and other resources.

A A fundamental principle that should guide the development of all educational programs, including those designed for gifted students, is that ample opportunities should be provided to allow students to realize their full potential. With respect to this principle, gifted programs differ from other educational programs only in that the application of the principle should lead to different educational provisions that reflect the fact that gifted students' learning styles and learning potential differ in many ways from those of regular students. (B) is incorrect because translating the affective domain into thoughts and actions is an objective that would apply only occasionally in specific situations; this is by no means a guiding principle of gifted education. Similarly, (C) is incorrect because the homogeneous grouping of gifted students in instruction is a desirable aim for some, but by no means all, instructional situations. Therefore, among the choices listed, this consideration is not the most appropriate guiding principle for the development of a gifted education program. Regarding (D), although concerns about financial and other resources do affect educational decisions to some extent, such concerns are not basically educational in nature and should not be the principle that guides the development of a gifted education program. In addition, (D) involves questionable reasoning: administering gifted education separately from regular education does not necessarily, or even usually, maximize available funding and other resources.

In general, most school curricula pay relatively little attention to the affective education of gifted students. Which of the following is the most likely reason for that inattention? (A) School staff tend to be more oriented toward meeting the cognitive needs of gifted students than meeting their affective needs. (B) The affective issues faced by gifted students tend to be virtually identical to those faced by their nongifted peers. (C) Studies indicate that school instruction designed to improve students' affective skills is generally ineffective. (D) The knowledge base on which to build affective education programs is as yet too limited to justify the expenditure of limited education funds.

A Historically, educators concerned with providing services to the gifted have attended primarily to the dimension that most obviously distinguishes gifted students from regular students; that is, their intellectual talents. The need to pay attention to gifted students' affective development is less immediately obvious and is only beginning to be recognized by educators of the gifted. (B) is incorrect because gifted individuals do not face affective issues identical to those faced by nongifted students. To the contrary, the gifted need to develop certain skills and understandings with regard to themselves and others that are a direct consequence of their being different from others in important ways. (C) is incorrect because researchers have, in fact, provided evidence for the effectiveness of various programs designed to improve gifted students' affective skills. (D) is incorrect for similar reasons: researchers and practitioners have built and continue to build a body of knowledge about the needs and characteristics of gifted students that is useful for the development of affective education programs for these students. Furthermore, the expenditure of limited funds to address only the cognitive needs of gifted students implies an unwarranted dichotomy between cognitive and affective education. Indeed, unmet affective needs may often prevent gifted students from deriving any real benefits from unidimensional instruction.

The norms for a standardized test are intended to help the test user to (A) interpret the meaning of the test results (B) administer the test the same way each time (C) correlate the results of the test with a comparable test (D) prescribe remedial actions in areas of need

A Norms provide a point of reference or comparison for analyzing the test results of an individual or group.

Which of the following best characterizes the study guide approach to compacting and differentiating instruction for a gifted fifth-grade student? (A) The student will work independently in the classroom to develop a deeper level of understanding about a particular concept. (B) The student will work in the library on a self-selected topic and present a weekly progress log to the classroom teacher. (C) The student will sign a contract to learn independently and will be excused from whole-class activities and classroom testing. (D) The student will work independently on a teacher-assigned topic and will learn to synthesize information from many different sources.

A Source: Teaching Gifted Kids in the Regular Classroom by Susan Winebrenner. Under the guidance of a teacher, the student is given alternate work in varying formats and can work at a relatively fast pace. (B) is not correct because the relationship of the library task to compacting/ differentiating instruction has not been clearly stated. (C) is not correct because it is not a given that the student is to be excused from instruction and testing; rather, only if the content were determined to be redundant for the learner would this be so. (D) is not correct because the gifted student would self-select a topic in light of personal interests.

An elementary school's new kindergarten teacher says to the gifted education teacher, "I have a delightful class. However, one student never stops asking questions, needs constant intellectual stimulation, and gets rambunctious when the class does anything that involves step-by-step processes. What should I do?" Given this preliminary information about the student, which of the following statements is the gifted education teacher's most appropriate response? A. "These characteristics sound like the developmental behaviors of a child who is gifted and talented. The student might benefit from differentiated instruction." B. It sounds like the student could be gifted. The situation might be helped if you redirect this energy by allowing the student to help others in class." C. "The student may be gifted but have poor social skills. Try providing the student with guided practice in interacting with classmates." D. "The student's behaviors indicate qualities associated with giftedness. Address the issue of curiosity by allowing the student to visit the school library on a regular basis."

A. At the elementary level, there are a range of behaviors that are associated with exceptional intellectual ability. A child with a strong sense of curiosity who "never stops asking questions" and "needs constant intellectual stimulation" is displaying two qualities highlighted in the research of Dr. Linda Silverman and others. A child's impatience with step-by-step processes may indicate a visual-spatial learning style, and may also indicate that the child masters material faster than his or her classmates. In this scenario, the most appropriate step for the gifted education teacher to suggest would be to recommend differentiated instructional opportunities as a method of engaging the student and gathering further information regarding the child's learning style, strengths, and needs.

Options for teaching gifted children may include: A. Group all ages together in one class B. Discourage extracurricular activities C. Require rote memorization D. All of the above

A. Because gifted children have advanced cognitive abilities and different educational needs, teachers need to develop lessons and activities to stimulate their active minds and inquisitive imaginations. Since most public schools are grouped by age rather than learning ability, bringing gifted students together in a single classroom no matter what their ages simplifies the teacher's job; he doesn't have to plan different lessons and activities based on intellectual need as he would in a heterogeneous classroom environment: Plus, like-minded students naturally stimulate each other. Accelerating a student to the grade level appropriate to his ability is another option. Testing a student on the subject matter before it is presented and then developing lessons and activities that fill in the gaps and challenge his preconceived ideas is another possibility. Allowing gifted students to take different levels of schooling at the same time and encouraging extracurricular activities are other options to consider.

1. Which of the following statements about gifted children is NOT true? A. Gifted children have fewer problems than other children. B. The intellectual and emotional development of gifted children may not be concurrent. C. Gifted children are more likely to be afraid of failure. D. Gifted children are prone to boredom.

A. Being gifted can, indeed, create a whole new array of problems for children.

Cognitive characteristics of gifted children include: A. Intellectual curiosity B. Problems concentrating C. Emotional rather than logical D. All of the above

A. Gifted children are found in all ethnic and social-economic groups. They have an exceptionally high level of intellectual curiosity with a need for precision in thinking and expression; they process data quickly and learn in an integrative, nonlinear manner by making intuitive leaps. At an early age, they show emotional sensitivity, empathy for ideas and people and concern for moral and existential issues. They have a highly developed code of ethics and an intense need for the world to be logical and fair. Gifted children are fascinated with ideas and words, have an extensive vocabulary and find multiple meanings in the simplest concept. They have the ability to think in the abstract, see all sides of an issue and offer logical solutions to complex problems. Their minds work in metaphors and symbols and they often have difficulty fitting in because they don't think the way other people do. They are frequently argumentative and have an idiosyncratic interpretation of events.

Based upon Karl's profile, which of the following adaptations would best support both Karl's demonstrated strength in reading and his development of social skills? A. including Karl in a group of advanced first-grade students who meet with third-grade students in a weekly literature circle B. giving Karl age-appropriate supplementary materials to read at home with his family C. providing Karl with vocabulary and word puzzles to complete independently during class reading instruction D. arranging for Karl to attend a fourth-grade class for additional reading instruction

A. In literature circles, small groups of students meet to discuss, respond to, and reflect on their reading. Placement with his most advanced age-mates and with students two grades ahead would be an appropriate match for Karl because of his high reading ability. In this format, Karl's academic confidence should encourage his active participation and provide a challenging reading environment. Listening to other viewpoints and learning to disagree constructively should support the development of his social skills.

Bonita is a kindergarten student who has been reading since she was three years old. Her parents are concerned that she will not be challenged in her kindergarten class during reading instruction. Which of the following types of assessment would be the gifted education teacher's best first step? A. testing Bonita's reading skills using above-grade-level texts and formative assessments B. having the school's reading specialist observe and assess Bonita's reading ability in class C. requesting an overall evaluation of Bonita's IQ before taking further action on her reading D. performing a miscue analysis while Bonita reads to assess accuracy and comprehension

A. Informal assessment of Bonita's reading skills using above-grade-level texts, reading program assessments, or formative assessments is an effective way for the teacher of a gifted and talented student to gauge reading level and comprehension skills. It is important to remember that a highly or profoundly gifted child may be able to read several years above grade-level. Assessment using above- grade-level texts will provide the teacher with the student's level of aptitude for higher level decoding skills together with comprehension of text and vocabulary. Differentiation of instruction for Bonita can then be designed for her specific strengths and weaknesses.

Which conclusion is supported by the pioneering work of Leta Hollingworth in gifted education? A. Highly intelligent children are extremely vulnerable and need emotional education. B. Equitable identification should be based on student performance rather than IQ testing. C. Leadership and creativity as well as intellectual traits can be indicators of giftedness. D. Artistic and creative abilities are neglected in favor of intellectual abilities in gifted and talented programs.

A. Leta Hollingworth's varied research in the field of psychology was extensive and groundbreaking for its time. She wrote the first comprehensive text on the gifted, Gifted Children (1926), detailing educational practices and social/emotional development for nurturing giftedness. Many of the theories she advocated are standard now in the field, and she was one of the first to study children with IQs over 180. To Hollingworth, the emotional growth and development of a gifted child was as critical as the cognitive development. She observed that highly gifted children were often socially isolated, unpopular, and unmotivated in school. The psychosocial development of the highly gifted child continues to be a relevant topic.

Which of the following statements best describes the primary focus of the most recent federal definition of giftedness (Javits Act, 1988)? A. identifying special talents and aptitudes that are context- and domain-related B. developing inherent intellectual and academic abilities that predict future achievement C. identifying exceptional general capacities for creativity and problem solving D. developing holistic ways to assess and nurture inherent gifts in all children

A. The 1988 Javits Act defines giftedness as, "Children and youth with outstanding talent who perform or show the potential for performing at remarkably high levels of accomplishment when compared with others of their age, experience, or environment. These children and youth exhibit high performance capability in intellectual, creative, and/or artistic areas, possess an unusual leadership capacity, or excel in specific academic fields." This definition shows that the broad range of giftedness relates to both domain (i.e., specific fields or areas of knowledge) and context (i.e., skills and abilities apart from, beyond, or across specific academic areas).

Jorge is an exceptionally gifted tenth grader who has a passion for literature. Jorge has written a novel, which he self- published and distributed at local bookstores, and he has participated in community literary activities. He is certain that he wants to be a writer. For the teacher of students who are gifted and talented, what would be the most appropriate strategy to adopt when Jorge seeks support in taking his ambition seriously and moving beyond high school? A. helping Jorge develop and refine a writing portfolio on an ongoing basis so that he can seek out opportunities to submit his work and participate in workshops as well as in college programs B. encouraging Jorge to become a literacy volunteer in an underprivileged community because this will help him present a well-rounded portfolio that demonstrates his connection to others C. helping Jorge start a writing blog and interview professionals in the publishing business as a way to make important connections that may help him get literary representation D. encouraging Jorge in his writing while realistically acquainting him with the difficulties even the greatest writers encounter and helping him focus on a way to make a living while he writes

A. This item requires examinees to apply knowledge of research-based and evidence-based strategies for fostering students' growth in personal competence and dispositions needed for exceptional academic and creative productivity (e.g., maintaining high expectations as evidenced in meaningful and rigorous activities; providing opportunities for self-exploration and development and pursuit of interests, strengths, and abilities; offering feedback that focuses on effort and perseverance). Professionals throughout many creative fields maintain portfolios showing samples of the quality and scope of their work as a way of presenting themselves for consideration for employment or sponsorship. Helping Jorge create and maintain a portfolio of his work will familiarize him with a process that can facilitate his advancement in his field. This approach puts Jorge in a position to seek and obtain opportunities to develop his craft in real-world contexts.

A gifted education teacher is constructing a summer science course in problem solving for gifted middle school students who demonstrate verbal and quantitative reasoning ability at least two years above grade level. The teacher plans to measure student achievement with an intensive final written exam and presentation. The teacher could most effectively use formative evaluation to promote student performance at advanced levels by: A. reviewing data on individual student performance as a basis to support giving competitive awards. B. studying systematically collected data on student performance to modify and revise the curriculum. C. analyzing data on low student performance to determine the need for reassessing any student's eligibility. D. collecting data on exceptional student performance to support a proposed course sequel.

B

An elementary school gifted education teacher is monitoring and evaluating activities in her advanced literature course for gifted fourth-grade students. As the year progresses, she systematically compiles and assesses samples of students' work from this course to create exemplars. These exemplars will serve as support data during a summer in-service, when the teacher will work with a fourth-grade general language arts teacher to compare the goals and outcomes of their programs. Which of the following best describes the purpose of the teachers' in-service task? A. scope and sequence development B. curriculum alignment C. single-subject differentiation D. curriculum compacting

B

Recent research has shown that gifted students from communities or cultures where their social identity may be threatened by high academic achievement most often benefit from opportunities to participate in: A. after-school enrichment programs that incorporate multiple academic content areas. B. multiracial and multiethnic talent development programs. C. intensified conflict resolution training paired with academic instruction. D. accelerated online academic instruction that minimizes social vulnerability.

B In order to maintain their identity with their primary group, students may underachieve or purposely avoid participation in gifted and talented programs and other programs if their culture or group is not represented in those programs. By creating and encouraging participation in multiracial and multiethnic talent development programs, educators encourage development of students' gifts and talents while respecting and integrating students' needs for maintaining their cultural ties and racial identity.

Elizabeth is a fifth grader who is gifted in language arts. She attends Mr. Watson's general education class for all subjects, and a gifted education specialist consults with Mr. Watson on how to differentiate instruction for Elizabeth. The class will be reading the first novel in a popular middle-grade adventure series based on mythological characters. Mr. Watson will focus on the writing element of Grade 5 Common Core Standard 9 that asks students to be able to "Draw evidence from literary or informational texts to support analysis, reflection, and research." Most of the students will be assigned to research and contrast the characters in a classical myth retold for children using the same mythological characters in the contemporary novel they are reading. Which of the following strategies should the gifted education specialist recommend that Mr. Watson use to make the task most appropriate for Elizabeth? A. allowing Elizabeth to read the entire series independently at her own pace, but to produce the same assignment as the other students B. offering Elizabeth readings of greater complexity, such as myths from other cultures, and a more challenging research question C. assigning Elizabeth the creative task of dramatizing the contemporary novel so that a portion of the story can be performed in class D. giving Elizabeth an examination on her knowledge of classical mythology that will exempt her from the unit and allowing her to create her own myths

B It is important to challenge students who are gifted and talented in ways that will extend their comprehension and provide appropriate challenge. Encouraging Elizabeth to read and evaluate myths from other cultures will give her ownership of her learning and challenge her to make intellectual connections as she analyzes and reflects upon the themes and elements of world myths.

Which of the following is a model curriculum for gifted education that focuses on enrichment for all students through high levels of engagement and the use of challenging learning experiences based on students' interests and learning styles? (A) Race to the Top (B) Schoolwide Enrichment Model (C) Mentoring Mathematical Minds (D) Depth and Complexity Model

B "The major goal of the SEM is the application of gifted education pedagogy to total school improvement. . . .SEM provides enriched learning experiences and higher learning standards for all children through . . . advanced follow-up opportunities for young people based on their strengths and interests [and] relating to learning styles." (A) is incorrect because this refers to a stimulus for improvement in schools endorsed by the federal government. (C) is incorrect because it represents a conceptually different focus. (D) is incorrect because the focus of this model is dissimilar to the intent of the SEM.

Students who score exceptionally well on IQ tests most often excel in which of the following areas? (A) Creativity (B) Convergent thinking (C) School grades (D) Critical thinking

B Intelligence tests consist mainly, if not exclusively, of questions to which there are specific correct responses. This type of question calls for a combination of memory and logical reasoning skills, a type of mental activity that is known as convergent thinking. Because students who obtain high scores on IQ tests have demonstrated, almost by definition, that they are good at convergent thinking, (B) is the correct response. Creativity, on the other hand, is almost synonymous with divergent thinking, which is the ability to generate novel, unpredictable ideas. Because individuals who excel at convergent thinking may have only average or low divergent thinking skills, a high IQ score is not necessarily associated with high creativity, and choice A is incorrect. Although intelligence test results often have a high positive correlation with school grades, it is not unusual for specific individuals to be very intelligent and yet earn undistinguished school grades. Because the association between IQ and school grades is not as strong as the association between IQ and convergent thinking skills, (C) is incorrect. Critical thinking, which involves evaluative skills, is not typically measured by standard IQ tests. As with divergent thinking, individuals may be highly intelligent without being skilled in critical thinking. Therefore, (D) is also incorrect.

Which of the following best characterizes the overexcitabilities of gifted students as observed by Kazimierz Dabrowski? (A) Antisocial behavior when frustrated (B) Supersensitivity to their environments (C) Obsessive-compulsive behavior (D) Intense interest in a complex topic

B Research contributions from the Polish psychologist Kazimierz Dabrowski (1902-1980) define overexcitabilities as "extreme intensities or sensitivities that affect the ways in which an individual experiences the world." Additionally, "Although most of us may have extra energy at times or have strong reactions to various stimuli on occasion, those with overexcitabilities experience these distinguishing behaviors regularly." (A) is not correct because antisocial behavior may not be characteristic of overexcitabilities. Similarly, (C) is not correct because obsessive-compulsive behavior may not be characteristic of excitabilities. (D) is not correct because while almost always welcome, intense interest does not necessarily accompany overexcitabilities.

Which of the following teaching approaches is most likely to enhance the creativity of gifted students? (A) Emphasizing concrete, real-life applications of topics rather than abstract concepts (B) Structuring assignments in ways that encourage divergent thinking (C) Accelerating coverage of required content to allow time for creative activities (D) Encouraging students to choose their own educational goals and to seek out their own resources

B The concept of intellectual creativity refers to the ability to generate new, unanticipated ideas and connections between ideas. This type of thinking is referred to as divergent thinking and is best promoted when teachers present students with open-ended questions, with problems that require new perspectives for their solutions, and with issues that invite a wide range of responses. (A) is incorrect because creative thought can be elicited by or applied to abstract concepts as well as to concrete real-life situations. (C) reflects a basic misunderstanding: creative activities should not be separated from regular school tasks and tacked on to the end of so-called regular tasks as a type of bonus. To the contrary, required educational content can and should be presented in ways that encourage creative thinking. (D) implies another basic misunderstanding: it is the responsibility of professionals, not students, to set educational goals. Furthermore, although students may demonstrate some creativity in identifying resources for specific tasks and projects, it is, again, the responsibility of educators to provide the basic resources required by students to attain educational goals.

A seventh-grade class is learning to solve simple linear equations using a function machine and graphing paper. Which of the following is an appropriate extension activity for a gifted student who has superior learning ability and is able to learn independently without following along with the regular classroom instruction? (A) Solving quadratic equations using tables of values and graphing paper (B) Solving systems of linear equations using a graphing calculator (C) Creating word problems that can be solved with linear equations (D) Creating posters illustrating basic algebraic concepts

B This activity would take the student to a level beyond linear equations using the latest technology to learn concepts and solve problems. (A) is not correct because it reflects a dated approach in that it does not use the most current technology available for the task. (C) is not correct because it does not challenge the gifted student to learn an important mathematics topic independently. (D) is not correct because this activity is not likely to challenge the student to use his or her talents to go beyond linear equations and learn about quadratic equations independently.

Ten-year-old Walter reads at the college level but will not sit still long enough to take a unit test in class. Walter invented a clothes-putting-away machine using bent wire hangers, old thread spools, and string, and he can intuitively grasp complex astrophysical concepts; however, he cannot explain his process when given a basic word problem in math. The first time he took an intelligence test, his score showed below-average ability. After he learned how to read, assessment revealed an IQ of 146. He has come to fear school because something always goes wrong, whether it be related to his erratic classroom behavior or his interactions with other students. The best provisional interpretation of the information provided by the teacher of students who are gifted and talented would be that A. Walter demonstrates indicators of being a math/science prodigy who has an anxiety disorder B. Walter shows signs of being twice-exceptional and needs further assessment for a possible attention disorder C. Walter is exceptional in both his asynchronistic behavior and giftedness, but he may have a depressive disorder D. Walter shows signs of sensory-processing issues and should be assessed for an autism spectrum disorder

B. "Twice-exceptional" is a term used to describe students who are gifted and talented and who also have a disability. Teachers of students who are gifted and talented should be aware that performance and/or behavior issues associated with a disability can often mask the student's giftedness. For example, Walter's erratic behavior and interactions are characteristics often associated with attention disorders. These characteristics may be interfering with Walter's ability to fully demonstrate his learning and achieve his potential.

Work products that can be included in a portfolio include: A. List of awards achieved B. Papers and presentations C. Written recommendations from previous teachers D. All of the above

B. A portfolio is a collection of the student's work assembled over a period of time (six week grading period, one semester, the entire year, etc). Various items can be include Contracts, copies of completed activities such as papers, presentations and pictures of props, performance assessments made by the student, peers and the teacher, copies of class work and homework, classroom tests and state-mandated exams. A portfolio is a powerful aide in assessing the student's progress and an excellent format to present to parents so they can review their child's progress. The decision on what to include should be a collaboration between the student and the teacher. What will be includeExamples of best work, worst work, typical work, some of each? Will the student keep a copy as a reference point? Decisions need to be made and rules established as early as possible in the process so progress is accurately and fairly recorded.

Tatiana is a sensitive, good-natured, seven-year-old who has been identified as highly gifted. Tatiana is quiet, reflective, does not eagerly participate in class, and prefers quiet and independent activities. She is happy at home, does well in her schoolwork, and enjoys her differentiated activities, particularly if they involve creativity. During a staff meeting, Tatiana's teacher mentions her traits to the school's gifted education specialist and asks, "Should I be doing more to help correct her tendency toward introversion?" Which of the following would be the gifted education specialist's most appropriate response? A. "We should not rule out learning disabilities. It is possible that her intelligence has masked the need for a further assessment." B. "Many children who are gifted and talented are introverted. Introversion is not a flaw, but rather an aspect of her personality." C. "She's still a little girl and she's very gifted. She needs patient guidance and she'll blossom into a more social individual." D. "Introversion is an aspect of asynchronous development. She will almost certainly outgrow it by the time she reaches young adulthood."

B. According to the Duke University Talent Identification Program's Digest of Gifted Research, the majority of gifted children are introverted, and because of their heightened awareness of themselves and others, they are also likely to be "sensitive." Parents and teachers can be supportive of an introverted child by being empathetic about feelings of shyness and respectful of his or her personality rather than trying to change it. A classroom climate that welcomes the diversity of all students' learning styles and academic strengths and needs may boost the student's involvement and social behaviors.

Eighth-grade science students will be studying Newton's three laws of motion. Two students in the class who are also in the gifted education program have experience with this topic. The gifted education teacher is collaborating with the classroom teacher to modify existing instructional plans to better promote the learning needs of the students who are gifted and talented while maintaining their involvement in the activity. To develop differentiated instructional plans for these students, the gifted education teacher should first: A. collaborate with the students who are gifted and talented in designing a timeframe that will guide their learning. B. pre-assess the background knowledge of the students who are gifted and talented relevant to the topic to identify and integrate appropriate extension activities during the unit. C. notify the parents/guardians of the students who are gifted and talented about adjustments being made to the instructional plans. D. establish a plan that will enable the students who are gifted and talented to complete individualized projects of their own choosing as extension activities.

B. All teachers are better able to differentiate instruction by pre-assessing the existing knowledge of the students so that instruction is appropriately challenging and meets academic benchmarks. In this example, since the gifted education teacher is collaborating with the general education teacher to develop a differentiated plan, an informal pre-assessment could be tailored to identify the extent of the students' content knowledge so as to inform instruction as well as plan differentiated extension opportunities.

Monique is a gifted high school junior who is considering skipping her senior year to start college early. She is most likely to be successful with early college entrance if she: A. holds leadership roles in several extracurricular activities. B. is a self-confident and independent learner. C. has an active, school-centered social life. D. is a perfectionist and highly competitive with peers.

B. As an early-entry college student, Monique would be entering an environment considerably more demanding, and with fewer support systems, than what she has been used to in high school. It will be largely up to her to determine how best to meet the new academic and social challenges she will encounter. Students who are independent learners recognize, maximize, and are confident in what works best for them in their learning process. Monique is most likely to be successful with early college entrance if she has the self- confidence that comes with strong independent learning skills.

Karl is a five-year-old student in a first-grade heterogeneous classroom. He is the second child of successful, outgoing parents and has one older brother. He has been assessed and identified to receive gifted education services. Karl is part of a group of gifted first- grade students who meet with the gifted education teacher for two hours each week. Academically, Karl is reading on a fifth-grade level and has an extensive vocabulary. In social studies and science, he shows an excellent grasp of abstract concepts. Although he is a year younger than his classmates, Karl is successfully working near the top of the first-grade math groups. In math, he particularly enjoys solving word problems, but he performs slightly below average on timed written quizzes of basic facts. Karl is quite shy and rarely initiates interaction with his classmates. When given the option, he usually chooses solitary play activities. In spite of his shyness and solitude, the classroom teacher has noted that he appears well-liked by classmates, and is sought out during free play. His fine-motor skills are behind those of the average five year old, and the art teacher describes these skills as being at the level of a four year old. Based upon this profile, which of the following conclusions would be most appropriate for a gifted education teacher to make about Karl? A. Karl should be paired with an extroverted student as a model to help him manage his social interactions successfully. B. Karl will need differentiated experiences to express his gifts effectively and overcome his shyness and delay in motor-skills development. C. Karl should be accelerated to a grade level closer to his reading level for effective development of his gifts. D. Karl's math abilities are below average when compared to his other abilities, and he will need individual tutoring to overcome this deficit.

B. Diverse needs and abilities require diverse educational options. Differentiation is the intervention that will best accommodate the range in Karl's learning profile and help him to progress in all areas. The gifted education teacher can tailor Karl's program so that differentiated learning experiences target his unique needs and abilities.

Gifted children typically get similar results with different types of I.Q. tests, such as the Wechsler Preschool and Primary Scale of Intelligence (WPPSI), the Stanford-Binet Intelligence Scale (SB), and the Wechsler Intelligence Scale for Children (WISC). a. Neither true nor false b. False c. True d. These I.Q. tests are not given to gifted students

B. False. While average and developmentally delayed children typically get very similar scores in a variety of I.Q. tests, gifted children more often get radically differing scores from test to test.

Behavioral characteristics of gifted children include: A. Difficulty understanding instructions B. Have to understand the how and why C. Frequently hyperactive D. All of the above

B. Gifted children are individuals. Some are outgoing and socially well adjusted and become effective leaders because they are able to earn the trust of their classmates. Others have trouble relating because they view the world differently than most of their peers. They have difficulty making friends and may become isolated and lonely. Each gifted child utilizes his intellectual curiosity in unique ways. However, they share common behavioral characteristics: They have an extreme need for constant, engaging mental stimulation. They get bored if information is presented in small segments or they are expected to learn by rote and repetition. They need to explore all aspects of a topic. They have to understand the how and why as much or more than the what. They have an insatiable curiosity about everything. They never stop asking questions. They have the ability to shut out all distractions and focus on a subject for long periods of time.

What areas are often considered when evaluating the contents of a portfolio? A. Intelligence B. Critical thinking C. Neatness D. All of the above

B. Once decisions have been made about what will be included, it is important to begin with baseline data for comparison as the portfolio grows. Selected material can be placed in a folder or large envelope with the student's name on the front. Each addition needs to be dated with an explanation attached stating why the item was included and what features should be noted. Teachers who use portfolios will often create assignments with the intention of including it in the package. As the contents grow, it may become necessary, because of space limitations, to review the items and remove some daily work and a few quizzes and tests. Once the portfolio is complete, the teacher needs to have a method to evaluate the contents and review the student's progress in areas such as creativity, critical thinking, originality, research skills, perseverance, responsibility and communication effectiveness. A checklist can be useful.

Lian is a gifted six-year-old student in Mr. Barker's heterogeneous second- grade classroom. She shows extremely advanced reasoning ability, especially in mathematics. Mr. Barker asks Lian whether she talks to herself when faced with a difficult word problem as she determines which operation to use. Mr. Barker encourages Lian to write down her problem-solving strategies revealed by this self-talk. This activity will most likely help Lian further develop her thinking skills by: A. accelerating her pace through the math curriculum. B. promoting her awareness of metacognition. C. providing her with opportunities for divergent thinking. D. increasing her achievement level on math tests.

B. Problem-solving provides particularly rich opportunities for developing metacognitive strategies. Following Mr. Barker's suggestion of writing down her self-talk will help Lian begin to monitor and evaluate her thinking and become aware of other basic metacognitive skills such as connecting new information to former knowledge and selecting thinking strategies.

Maxine and Iris are ten year olds who are being considered for gifted education services. They have both been nominated by their classroom teachers. Maxine Current Grade Placement 5th Overall Achievement Test Results (Grade Equivalent Scores) Reading/Lang. Arts: 6.3 Mathematics: 7.0 Broad Achievement: 6.6 WISC-III Verbal IQ/Full Scale IQ: 127/135 Extracurricular Activities avid stamp collector, creative writing, photography Iris Current Grade Placement 5th Overall Achievement Test Results (Grade Equivalent Scores) Reading/Lang. Arts: 8.2 Mathematics: 6.7 Broad Achievement: 7.8 WISC-III Verbal IQ/Full Scale IQ: 132/122 Extracurricular Activities voracious reader Which of the following preliminary conclusions could be drawn based upon the above information? A. Iris probably works very hard to keep her test scores high. B. Programming for Maxine should feature enrichment activities focusing on her strengths and interests. C. Iris is appropriately placed in fifth grade. D. Maxine's preferred activities are solitary pursuits, so she will need to develop her interpersonal skills.

B. Since Maxine is a fifth-grade student whose broad achievement grade equivalent is 6.6, effective enrichment focusing on her strengths and interests would broaden and deepen her learning experiences by allowing her to discover information and ideas that are beyond the scope of the general curriculum. Appropriate enrichment for Maxine should emphasize qualitative development of thinking skills rather than quantitative accumulation of facts.

Stella, a gifted 14-year-old, is the first member of her family to be fully bilingual in both English and her home language. Her family is extremely proud of her hard work and good grades in school, but they are uncomfortable with enrichment activities that take her out of her home community. Stella's loyalty to cultural expectations and her need to belong make her feel conflicted about the possible consequences of her exceptional academic abilities. Research indicates that Stella could best be helped to achieve a desirable balance through her teachers' implementation of which strategy? A. arranging for Stella to receive counseling from a trusted adult that directs her attention to the innumerable contributions of immigrants to all areas of U.S. history B. supporting Stella's strong sense of her own heritage and identity while facilitating her participation in the larger multicultural society C. arranging for Stella to participate in gifted programs designed specifically for students with ethnic and cultural identities similar to her own D. supporting Stella in considering diverse scenarios that include the possibility that her intelligence can be applied in many ways that do not involve higher education

B. This item requires examinees to apply knowledge of effective strategies for responding to the concerns of students who are gifted and talented and their parents/guardians. Recent research indicates that gifted and talented students from culturally and linguistically diverse backgrounds are more likely to thrive if they are provided with culturally responsive curriculum and instruction that enable the students to connect academic experiences with their lives outside school. Opportunities to interact with role models who share cultural or linguistic backgrounds with the students promote their cultural identity as they develop social and academic skills that prepare them to participate in a multicultural society.

Ms. Pitt, a second-grade teacher, is exploring the history of the school's rural Hudson River community with her mixed- ability class. Angelina, a highly gifted seven-year-old in the class, has a passion for local history and has become an expert on the subject. Angelina's grasp of the material far exceeds that of her classmates, and she often asks questions considerably beyond the scope of planned lessons. While Ms. Pitt wants to include Angelina in thematic class discussions, she also wants to differentiate instruction to meet her intellectual needs. What would be the best advice a teacher of students who are gifted and talented could give to Ms. Pitt regarding how to proceed? A. "Provide opportunities for Angelina to apply her extensive historical knowledge by having her design enrichment materials and activities for classmates." B. "Help Angelina define a research question and a final product that will allow her to pursue her interests at an appropriately challenging level." C. "Give Angelina repeated opportunities to share her knowledge of history with others by having her act as your teaching assistant during certain lessons." D. "Allow Angelina to independently seek out the answers to any questions that occur to her and to report her findings to you at the end of each week."

B. This item requires examinees to apply knowledge of research-based and evidence-based strategies for selecting, adapting, and differentiating curricula to incorporate advanced, conceptually challenging, in- depth, distinctive, and complex content; strategies for selecting curriculum resources and product options that respond to students' cultural, linguistic, and intellectual differences; and strategies for using metacognitive models to meet the needs of students who are gifted and talented. The best practice in the differentiation of instruction for gifted and talented students is to use the general curriculum as a foundation, but provide opportunities for students to select related topics to explore in-depth. The research question Angelina defines should address higher-order thinking skills such as analysis and evaluation. Providing Angelina with the opportunity to choose a topic and product of interest to her will enhance her engagement with the content.

Examples of work products include: A. Home videos B. Bulletin boards C. Art work from childhood D. All of the above

B. Work Products are completed assignments that are evaluated on the topic chosen as well as creativity, originality, organization, understanding of the subject matter, social and academic progress and success in meeting and/or exceeding predetermined criteria; plus any other items deemed important by the individual teacher. Work products can take many different forms, including but not limited to; research papers, poems, fiction and non-fiction stories, bulletin boards, video and audio tapes, computer and laboratory demonstrations, dramatic performances, debates and oral presentations, paintings, drawings and sculptures and musical compositions and performances.


Related study sets

COMD 2050- Chapter 14 (Second Language Acquisition and Learning)

View Set

Chapter 7 Overview of the Financial Audit Process

View Set

Activités fonctionnelles - Examen 1

View Set

Facts about Martin Luther King Jr.

View Set

The Roman Catholic Church in Medieval Europe

View Set